33 posts / 0 new
Last post
arogers
arogers's picture
Newton's Third Law

I am interested in conducting some personal research on Newton's Third Law of Physics and its application to how body structure is an important part of a powerful strike.  My interest is in broadening my intuitive understanding of this concept so I can better explain it to students.  In other words, I can feel it and have no doubt about the importance of good structure but want to be able to explain it clearly at a more theoretical and detailed level. Most of the online resources that I have been able to locate seem to equate Newton's Third Law with either the collision of forces moving in opposite directions or the pulling back of hikite to generate forward force in the punching arm.  Both of these miss the point for me, which is how a strong structure can facilitate a stronger technique by being better able to absorb the resulting force of a strike. 

Can anyone recommend some good resources to review on this point?  Chris Denwood covers it in his book on Naihanchi which I have read but am looking for other sources.

Many thanks

Chikara Andrew
Chikara Andrew's picture

arogers wrote:

I am interested in conducting some personal research on Newton's Third Law of Physics and its application to how body structure is an important part of a powerful strike. 

I must confess not to be a great expert on this but have engaged in this discussion before and I'll add my own thoughts on why we should be aware of it when teaching karate. However a quick search for Newton Laws and Punching brought up this link which has a useful discussion on the matter and also one of the answers also introduces the key points of Newtons Second Law, it is too simple to consider the Third Law in isolation as there are many other factors:

http://physics.stackexchange.com/questions/213594/how-does-newtons-third...

The original question posed above highlights the reasons why it is important for us to consider and understand the laws of physics and how they relate to training. Punching fresh air is fundamentally different from punching an object. There are three phases to a punch, the wind up, free flight and impact. Only the wind up is the same, there can be no free flight of a punch, strike, kick, whatever if there is no target on which to impact there will always be an element of control by the opposing muscles to prevent loss of form or overextension/injury. 

I know a lot of karateka who have great looking technique but are unable to generate real impact.

arogers wrote:

Both of these miss the point for me, which is how a strong structure can facilitate a stronger technique by being better able to absorb the resulting force of a strike. 

I'm not quite sure I understand what you mean by this, is it that by having a stronger body then you will be able to produce a stronger punch?

Whilst physical size is a factor, mass, so is acceleration, often larger people move slower. With training a smaller person of slighter build should be able to achieve greater impact with a punch that a larger assailant who has little or no training. This is one of the key elements of karate training, trying to transmit the maximum about of force into an opponent with every strike. 

arogers wrote:

Most of the online resources that I have been able to locate seem to equate Newton's Third Law with either the collision of forces moving in opposite directions or the pulling back of hikite to generate forward force in the punching arm.

If I understand the point above correctly it is referring to the hikite proving the "equal and opposite" force which is incorrect, I think this comes from those that pivot around a central axis where the hikite and its corresponding hip is driving backwards in order to drive the punching arm and hip forward. I am firmly of the double hip school and therefore there is nothing going backwards and in terms of delivering a punch the I don't really consider the hikite. It is a useful tool early on to help students increase acceleration, but over time there isn't a need for this additional aid.

Andrew 

Iain Abernethy
Iain Abernethy's picture

arogers wrote:
Both of these miss the point for me, which is how a strong structure can facilitate a stronger technique by being better able to absorb the resulting force of a strike.

I think you’re misrepresenting the Newton’s Third Law in the way you are presenting it.

The law states that, “For every action, there is an equal and opposite reaction.” This means that we can think of a collision of two objects (say a fist and a face) as being a pair of forces. These forces are equal but opposite in direction.  So at the moment fist hits face there is the force going into the face (which is why it gets damaged) and there is the same level of force going into the fist (which is why it can also get damaged if incorrectly formed). However, nowhere in the law is there any inference that the actual level of force can be made unequal (i.e. more into the target). It's always equal.

Think of two pool balls colliding. If I fire the cue ball into one of the other balls, at the moment of impact, Newton’s Third Law will apply. Just about the worst shape you can imagine for absorbing force is a sphere. Very little of the sphere is in contact with the table and hence there is very little friction. A sphere is very easy to move as a result. When the two balls hit, we know from experience that the cue ball will slow down, stop or bounce back a little. The other will move forcefully away. The two balls do not leap away from one another at equal speed. One has momentum and one does not. Therefore, the equal and opposite forces slow the cue ball down, and accelerate the struck ball (Newton’s 1st law doing its thing). Most of the “back shock” from the struck ball is taken up slowing down the cue ball; which is why they don’t shoot apart at equal speed. It’s also why we don’t need to concern ourselves with “bracing for the back shock” when striking because the vast majority of the “back shock” will be “absorbed” by the deceleration of the strike (Newton's 1st Law). You simply don’t need to the body “locked and ready”. There are other problems with the proposal too.

A strong punch, as we know, will drive through the target. You are making the assumption of a target that returns 100% of the force. In reality, the target will be deformed by the force. If you punch a brick wall you hurt your hand a lot more than if you hit a soft pillow. Newton’s Third Law is in effect in both cases, but the reason far less force comes back into your hand when hitting the pillow is because the force is dispersed into the pillow. People are also soft in places, and they move a lot too. If you hit the head (which is hard) it will move away from the fist; like the struck cue ball. The body does not need to be in a position to absorb force (although the fist, as the impacting tool, will).

The main reason this misunderstanding about the need to absorb “back shock” persists is because of the Markiwara. The makiwara is nothing like hitting a person. It is more rooted and fixed than a person will ever be; and therefore you do feel more force coming back due to the totally unrealistic integrity of the object. If people fail to appreciate that, and falsely believe the makiwara to be a good representation of real world striking, then they will shift their striking method accordingly. The posture and motion becomes one of “absorbing force” as opposed to “generating force”. That can actually result in weaker punches. Don’t get me wrong, the Makiwara has its place, but to base your striking methods on it alone is very problematic because it is nothing like hitting an actual human being.

What we have at impact is this:

FORCE X > < FORCE Y

where X and Y are equal in magnitude, but opposite in direction.

What we need for maximum impact is a motion and posture that will increase the X to be as high as possible. A motion and posture that will resist Y as much as possible will be entirely different. It is the difference between throwing an object and catching it.

We see this all the time in physical activities. Think of the end positions of baseball pitchers, shot-putters, tennis players on a serve, etc. None are positions that can absorb a great deal of force. They are, however, positions that generate a great deal of force. That’s what is needed when we strike. A motion that delivers maximum force and will drive through the target (in a tactically sound way). Newton’s Third Law will apply to the integrity of the striking tool, and it is what will cause the strike to decelerate. It’s a mistake to think the whole body needs to be in a position to absorb the entire force of the strike coming back though. If that was the case the Superman punch would be incapable of hurting anyone.

It’s a fundamental misunderstanding of physics to say that a structure’s ability to absorb force is the same as its ability to apply force, or that we can somehow create force by being prepared to absorb it, or that we can send more force forward by repelling the back shock back into the target (that would break Newton's Third Law because the forces would no longer be equal and opposite).

arogers wrote:
I can feel it and have no doubt about the importance of good structure …

When it comes to impact, “stances” are not about absorbing force, but generating it. This article may help:

https://iainabernethy.co.uk/article/my-stance-stances

arogers wrote:
how a strong structure can facilitate a stronger technique by being better able to absorb the resulting force of a strike.

It can’t. It’s a misunderstanding of Newton’s Law, the reality of striking, and the purpose of stances. You can’t generate force by being able to absorb force. That can be seen it pretty much any physical activity you can think of. It ignores the reality of striking i.e. people move and break when you hit them; they are not solid unmoveable objects. What you need to focus on is how your movement and posture can generate force. Remember that people can generate knock-out power when in the air or on one leg. The fact that can be observed should put this misunderstanding to bed.

Look at the photo of this superman strike. He is not in contact with the floor and hence if he were to hit something totally immoveable he would be propelled backward. Thankfully he is hitting a person, and because of the fact people are very moveable he does not need to worry about his ability to absorb the returning force. He’s moving in a way that will generate huge forward force; the fact that same position is not suited for absorbing force is an irrelevance. The back shock simply decelerates the strike, while the target is displaced / damaged by the forward force.

arogers wrote:
Can anyone recommend some good resources to review on this point?

I’d leave it alone from that angle. Fighting is about movement, not being static. The only static things are unconscious or about to be unconscious. When it comes to impact, stances are not about “structure” but moving bodyweight. Power is about generating force, not absorbing force.  

When we are grappling (i.e. pushed or pulled) then we need the ability to absorb force and have structural integrity; but if we are talking about striking, then Newton’s 3rd Law does not apply in the way suggested.

Newtons Third Law applies to striking motions though. As an example, if I push with my back leg then a force will be sent into the earth. Because the earth is solid (and hence does not displace the force) the resulting equal and opposite force propels me forward. That forward motion can be used to get power into my strike. It’s the same way the starting-block helps sprinters get a fast start: they push into the blocks – which are fixed – and the equal and opposite force pushes the sprinter forward.

So In summary, I think you are barking up the wrong tree in trying to apply Newtons Third Law to striking in the way that was suggested. It’s not right in theory or practise and I’d shift your focus to generating power through dynamic motion. The linked stances article has lots of historical and practical examples of how “stances” truly generate force. I hope that helps.

All the best,

Iain

arogers
arogers's picture

Terrific responses, thanks to both of you.  I can see that I am barking up the wrong tree, and also that my use of a makiwara has led to incorrect assumptions on my part.  I actually do focus a lot on moving bodyweight and generating force, so my question was intended to get more understanding of a different aspect of striking.  And now I have it, but not what I had expected!

Just for completeness, I wanted to clarify that what I was attempting to describe in my original question was whether a weak structure upon impact could cause you to "bounce" and result in loss of force through the target.  For example, if your elbow buckled the moment you struck someone in the head.  Perhaps again I am misled by the unnatural firmness of the makiwara and many years of punching large people in the stomach not the head.

Chikara Andrew
Chikara Andrew's picture

arogers wrote:

Just for completeness, I wanted to clarify that what I was attempting to describe in my original question was whether a weak structure upon impact could cause you to "bounce" and result in loss of force through the target.  For example, if your elbow buckled the moment you struck someone in the head.  Perhaps again I am misled by the unnatural firmness of the makiwara and many years of punching large people in the stomach not the head.

You are correct here that the alignment of the fist, arm, body have a direct effect of the impact. If there is a weakness in the structure used to deliver the force, lets say the punching hand, then some of that force will dissipate and not be transferred to the target. One of the most obvious examples of this you see when introducing students to impact work is the wrist, if the wrist rolls, not only does it cause injury to the striker but the impact of the strike is greatly reduced. 

I think it was Funakoshi in Karate Do Kyohan (I don't have the book to hand) when talking about targets simplifying striking as using a hard part of your body to strike a weak part of the opponents body. 

That hard part needs to be conditioned and trained to withstand the impact in order to do this as efficiently as possible and this is why I made the reference to fresh air as opposed to impact training.

In simple terms force equals mass times acceleration but real life is not a physics experiment, there will be other variables involved, some under our control and some not. All the opponent has to to is move away from the strike and its effectiness is reduced.

Andrew 

Iain Abernethy
Iain Abernethy's picture

arogers wrote:
Terrific responses, thanks to both of you.

My pleasure! Pleased it helped. Thanks for kick off an intresting thread!

arogers wrote:
Just for completeness, I wanted to clarify that what I was attempting to describe in my original question was whether a weak structure upon impact could cause you to "bounce" and result in loss of force through the target.  For example, if your elbow buckled the moment you struck someone in the head.

I’m with you and I think that is partially valid. A bent wrist will buckle on impact, because of Newtons Third Law, and hence the forward force into the target will be greatly reduced. However, the key thing remains that the posture and movement generate huge forward force. We should not think of making the entire body position one that resits force, because that will be very different from one that generates it (as per the examples in the post above). If the forward force is great, it will drive into the target, displace it, move it, or break it. It won’t bounce off, because the forward force is still there driving the weapon into the target.

arogers wrote:
Perhaps again I am misled by the unnatural firmness of the makiwara …

That definitely happens. Karateka start training in the optimum way to hit a Makiwara, and not the optimum way to hit a person. People move and morph in a way the makiwara does not, and hence the anticipation of a back shock that never comes can become an impediment of optimum striking.  

arogers wrote:
… and many years of punching large people in the stomach not the head.

I don’t think that matters. If you look at the pictures below you can see that the body morphs when hit (unlike the Makiwara). The body is not a solid object that will return force with great efficiency.

Additionally, the posture when delivering such body shots delivers great forward force, but the same posture would not be great for receiving force.

By way of another example, if I was pushing a car while facing forward (I know for safety’s sake you are supposed to put your back against it and walk backward into the car, but run with it for the sake of this illustration) I would have my hands on it, leaning forward, feet pointing forward, knees pointing down, legs bent and driving forward, and heels off the ground. That is the posture and motion I would use to move the car forward. Now, if the car was rolling back toward me, such that I now need a posture to absorb the force, I would turn sideways on, put my shoulder against it, turn my feet to the side, have my back leg locked and with my knees to the side. Newtons Third Law applies in both, but the “receiving posture” would be very ineffective as a “pushing posture” and vice versa.

So back to striking, if we confuse “generating” with “absorbing” we will be utilising the wrong posture and way of moving. The makiwara has its place, and Newtons Laws of motion can be useful when analysing movement (one of my teachers regularly states that Newton was the greatest martial arts who ever lived for just that reason!). However, a makiwara is not a good facsimile of real world hitting.

Additionally, Newton’s Third Law does not state in any way that there is a way to “unbalance” the EQUAL and opposite forces (i.e. 70% into enemy, 30% into you) through an ability to resist the back shock. It would actually be a violation of Newton’s laws if you were able to resist or redirect the force in that way. That’s the whole point of the law! The forces are ALWAYS equal.

What is needed is the biggest possible forward force so the target is displaced of destroyed. Talk of absorbing, resisting or redirecting the back-shock due to Newton’s Third Law in order to generate a harder strike, while sounding scientific, is actually in violation of the very law being quoted.

Made simple, move in accordance with Newton’s SECOND Law if you want power! Get the maximum bodyweight into the technique with maximum acceleration. Tactical considerations always factored in too of course.

All the best,

Iain

manusg34
manusg34's picture

Great discussion! Like all of you guys I grew up in karate thinking makiwara was king. But through training saw it had its flaws. So I got away from training with itand started only really using fucus mitts. But a few months later while breaking up a fight in my bar I sprained my wrist and cut my knuckles on someone. So after I was healed I put makiwara back into my traing but only as a part of it still heavily using the focus mitts too. The "Board of Wisdom " (makiwara) I use to condition my hands and the technique of my punch. It seemed to have worked for me have not hurt my wrist in years hahahah.

Matt

Marc
Marc's picture

Newton's third law ist the physical background of the following wisecracks:

"When Chuck Norris does a push up, he isn't lifting himself up, he's pushing the Earth down."

"Throwing is the art of hitting people with planets."

What both are referring to is that when a body exerts a force of magnitude F onto the planet, the planet exerts a force of the same magnitude F in the opposite direction onto the body.

Three examples to illustrate the idea:

1) When you put a brick an a table, the force of gravity pulls the brick towards the center of the earth (commonly known as downwards). The fact that the brick does not fall through the table into the ground shows that there is another force of equal magnitude that's pushing against the brick from below, keeping the brick in place. This opposite force has to do with the physics on the atomic scale that keeps objects from passing through one another. Ask a physicist for the details.

2) When you blow up a ballon and tie it, the air that you blew inside pushes the skin of the ballon outwards, stretching it, while the skin of the ballon would rather like to collapse again, thus condensing the air inside it. As long as no energy is lost (heat, a leak), the two forces stay in equilibrium, which is why the ballon keeps its shape.

3) A spacecraft needs to change course to the left. It burns some fuel and blows the exhaust gas out to its right-hand side. While the gas is pushed to the right, away from the spacecraft, the spacecraft at the same time is pushed to the left, away from the gas, with the same amount of force.

Note that we are talking of the same force, not necessarily about the same speed or accelaration.

Read more on Newton's third law at: http://www.physicsclassroom.com/class/newtlaws/Lesson-4/Newton-s-Third-Law

Iain in his post illustrated how this principle applies to generating force for a punch by pushing your back foot backwards against the ground which will propell your body forward thus adding energy to your punch. Beware of slippery grounds though, friction is necessary for this to work.

Iain Abernethy wrote:
Newtons Third Law applies to striking motions though. As an example, if I push with my back leg then a force will be sent into the earth. Because the earth is solid (and hence does not displace the force) the resulting equal and opposite force propels me forward. That forward motion can be used to get power into my strike.

As to your question:

arogers wrote:
how a strong structure can facilitate a stronger technique by being better able to absorb the resulting force of a strike.

In physics terms you really to not want to absorb any of the energy. Instead you want to transfer as much energy as possible into your opponent.

Concerning structure/posture, you want as much of your body mass to contribute to the impact on your opponent with as much speed as possible. Consider a forward kick (mae-geri): If you'd lean back while kicking, you'd take the mass of your upper body out of the equation. You might even bounce yourself off your opponent. Instead keep your body upright and push yourself forward (including the upper body) with the supporting leg. Your kick will have more power. The same is true for the hip: When you turn one side of the hip away from the target you take body mass away from your punch.

About the problem that you might hurt your hand while striking your opponents head: Newton's third law still applies, but it is less relevant because other aspects of physics come into play here: Inelastic collision, deformation and mass inertia.

So the when your fist F collides with your opponent's head H, then F exerts a force onto H while H exerts a force of equal magnitude onto F. The interesting part is what the forces do to F and H respectively.

Some of the kinetic energy of F will be transferred to H. But not all of the transferred energy will stay kinetic and set H in motion. Some of it will be absorbed in the form of deformation. When part of the total kinetic energy is "lost" in another form of energy we speak of an inelastic collision.

To what degree F or H will be deformed depends on how strong or weak their structure is. So make a tight fist.

Another thing to consider is that how you apply force to an object has different effects on that object. Imagine applying force to an open door (in the direction that would close it). Think of three different ways to do that:

a) Put your flat hand on the door and then push heavily. The door will slam close.

b) Hit the door with your flat hand at high speed. The door will not close but stay more or less in place and rattle and jolt, maybe even the hinges will take damage.

c) Take a gun and shoot at the door. The door will not move much but the bullet will create a hole in it.

So Newton's third law is at work when we strike but it doesn't tell us much about what actually happens at impact.

Take care everybody

Marc

Marc
Marc's picture

Braniac's "Things but very slowly" has a nice video on the impact of a hand to a face:

Stevenson
Stevenson's picture

This is a subject that has been of enormous interest to me over the years being a science nerd.

I think people get caught up too much in the third law ie F=ma - where F = force, m = mass and a = acceleration.

What really matters is not just the force, but impulse, the amount of Force over time ie I = F/t. That's the amount of force directed into an object over time. Also, elasticity and force over an area described as pressure in physics, and momentum. They are all factors.

I think what the OA is looking for is a way of describing why good structure is important in delivering a techique using Newtonian physics. There is an excellent if very old scientific paper on this I read years ago - but I can't for the life of me find it - describing the difference between two punching methods. But I'd like to throw my 2 cents in to how to think about force within the context of teaching a karate punch:

- When you punch an object or person, you build up momentum with your punch. (p=mv)

- When it connects, energy is conserved by accelerating the target and deccelerating your "punch". How much each experiences depends on mass.

- The body or object you hit has "momentum" too, but because it's not moving we more properly describe that as "inertia".

- But if you only punch with your arm you only have the weight of your arm to deccelerate. In order for it to translate into enough force to move a body it would have to be going fast - faster than you can probably punch.

- But if you can slow down the rate at which it decelerates, you will transfer more force into the object. This is impulse - you continue to apply force during the moment of impact. (This is where it can get messy because the "moment" will be dependant on the elastic properties of the object and it's inertia.)

---

- Another way to put it is; if you apply force to something with inertia, you will feel that inertia slowing your fist down as you make contact.

- Because you are trying to punch through (force over time) your own inertia will be affected, you effectively "push" yourself off the object.

- If your structure is correct you can insure that the energy you are using to move your object is going towards moving the object and not you as well.

---

Kris Wilder has a nice way of putting this; in karate we try to use structure against strength - our bones against their muscle. We look for ways energy can "leak" from weaknesses in our structure. For example, if a shoulder is not locked in at the point of impact, then energy will "leak" from that point and not be directed into the target.

Zach Zinn
Zach Zinn's picture

I'm a huge fan makiwara training still, I think it does something that a heavy bag can't, and vice versa. Personally, both are integral to my understanding of striking hard. The point of a makiwara though is not to be a "realistic target" (talk about a lot of variables), it's to phsyically force habits of good structure over time. On that subject though, IMO the best "realistic target" if you want to test that is a guy in body armor.

" Both of these miss the point for me, which is how a strong structure can facilitate a stronger technique by being better able to absorb the resulting force of a strike. "

This seems backwards to me..you want to -not- absorb the power of the strike, but to put it all into the target, is that what you are saying?

arogers
arogers's picture

 

Thanks again to everyone that has chimed in.  Stevenson hit the nail on the head with exactly what I was getting at, particularly the last bullet point - ensuring that as much of whatever power you are able to generate is transferred into the target as possible.  That is the type of "penetrating strike" that I understand (and believe) a makiwara is intended to help develop.  I missed the point in Iain's answer about one ball accelerating and the other decelerating on the first read, but I can see now that he touched on the same point.  In that example, I would say that the cue ball could be said to have the type of solid structure that I was trying to refer to.  A useful example that incorporates variations in the quality of structure analogous to a punch might be a comparison of theoretical car crashes, one involving a vehicle engineered with a crumple zone (i.e., poor structure) and the other involving an otherwise identical vehicle that does not have one (i.e., good structure).  Assuming all other factors are equal including identical mass and velocity at impact, the one with the crumple zone is going to do less damage when it hits something, by design.   

I think I have what I was looking for, but if anyone else has anything to add thanks in advance.

Iain Abernethy
Iain Abernethy's picture

Stevenson wrote:
We look for ways energy can "leak" from weaknesses in our structure. For example, if a shoulder is not locked in at the point of impact, then energy will "leak" from that point and not be directed into the target.

I get and agree with the sentiment beneath this, but this is not to do with "structure" but with the efficient generation of power. I can shift my weight and rotate my hips, but if the arm motion does goes provide an efficient way of transferring the bodyweight shift, then it will be a weak punch.

It’s not a structural thing, but a mechanical thing.

I dislike the word “structure” in this context because it infers something static. A structure is built to withstand forces (building, bridge, tower block, etc.) not generate them. If we think of things that generate forces (motor, bullet, hammer, engine, etc.) there is a lot of motion happening. Back to the initial point, when it comes to striking, your ability to withstand force has little to do with your ability to generate force.

We can incorrectly think of “stances” as being static instead of snapshots of motion i.e. places we move to and through. Standing in a static position (“a solid stance”) in front of a totally static target like the makiwara, which will send a good deal of force back into the body due to how unrealistically solid and fixed it is, gives a way of striking that is antithetical to real world combat and impact. It needs to be suplimented with more realistic impact equipment.

I also dislike the word “locked” (i.e. locking the shoulder) because this again infers something static. “Correctly aligned” would be a better way to put it. We want motion to generate power. We also want motion combatively because, as I often remark, there are only two unmoving things in combat: the unconscious and the about to be unconscious.

We want to move like a powerful machine (i.e. be mechanical and moving) and not think of being a solid structure i.e. static.

Other striking arts never talk of dealing with back shock in this way (because they don’t have the tradition of the makiwara?). They move in a way that generates power and they don’t think of standing statically in a way that absorbs power. If you read the words of the past masters they were clear “stances” were not supposed to be structural and static, but mechanical and moving. No one should try to use Newtons Third Law (which in truth disproves the point they are trying to make) to justify this misunderstanding that arose from the 3K era of karate.

The fabled “back shock” that we need to brace the whole body for doesn’t happen in reality because that back shock is taken up simply slowing down the strike.

Newton’s 1st Law states that:

“An object will remain at rest or in uniform motion in a straight line unless acted upon by an external force.”

The “external force” that slows the punch is the collision from the impact (the equal and opposite force identified in the third law). Much of the talk on this (not in this thread but elsewhere) is ignoring the fact that the punch has a positive force prior to the collision. It’s a fundamental mistake to think of the impact as an instantaneous event generating two forces shooting away from one another; one of which we need to be braced to absorb. That's not how it works.

Like the pool ball analogy earlier, we know the cue ball (a ball, which is not a stricture great at resisting force) does not shoot away when it hits the other ball (putting aside spin). It stops moving while the other ball shoots away.

You get the same thing in a Newtons Cradle too. The ball(s) stops moving on impact. It does not shoot backward! It’s a ball on a string, which is not a locked “structure” capable of absorbing force. It does not need to though because the equal and opposite force from the collision brings it to a halt: it does not throw it backward. That would need an UNEQUAL and opposite force i.e. be in breach of Newton’s Third Law. Becasue the force is equal and opposite, there’s nothing for the ball to resist. That’s why it stops still.

I can’t understand why karateka can think their strikes are somehow different from these fundamental laws in action. Especially when the fact we don’t need to a solid structure to deal with “back shock” is readily observable. We (should) know this from experience of striking.

We don’t need a structure capable of absorbing force. We need motion capable of delivering force.

Let’s say a fist is moving forward with a value of +X. At the moment of impact +X will be delivered into the target. Newton’s Third Law means that -X (an equal and opposite force) will be sent into the fist. The fist had a value +X and the impact sent -X back into it. +X added to -X = ZERO. So there is no force sending the fist backward (as with the balls in a Newton’s Cradle).

So why do we need a locked structure to deal with zero force? We don’t! We just don’t.

That’s why the ball on the newtons cradle stops dead (a positive force added to an equal and opposite force = zero). It’s free hanging ball on a bit of string (a weak structure) but it does not jump away. If you want the target ball to be hit harder, then swing the ball harder (the ball will still stop dead because the greater impact generates a greater equal and opposite force).We don’t need to sure up the impacting ball and string into a more solid structure to get more impact. Punches are no different. Newtons third law (and his cradle) proves such a structure is not needed; so it is completly wrong to quote it as proof we do.

It’s pseudoscience being used to justify a form of practice based on a fundamental misunderstanding about the nature of stances and what is truly required to hit hard.

It’s an falsehood to stay we need a strong structure to hit harder, and then to quote Newton’s Third Law as the reason. Newton’s cradle shows that’s not the case. Indeed, real world punching shows that’s not the case.

We don’t need a locked structure to deal with the back shock in order to hit hard. We need to move efficiently to generate force. Indeed, that may be the best way to summarise it:

We need mechanically efficient motion, not static structure, to generate force.

All the best,

Iain

JWT
JWT's picture

Very interesting discussion.

Linked to this, although we were originally chatting about the structure of the fist after I watched a video that incorrectly explained different fist structures for Uke and Tsuki techniques (the instructor had them the wrong way round), Karate Dr and anatomy professor R Gutierrez made this video for me.

Hope it helps

John Titchen

Iain Abernethy
Iain Abernethy's picture

That’s an interesting video John. Thanks for sharing. I agree with him about the “tensing on impact” problem, but I think it has its origins elsewhere.

If you do a lot of punching the air you are going to hyper-extend your joints because there is no impact to decelerate the punch (Newtons First Law). So we have to decelerate our punch ourselves. To keep the punch fast, we need to decelerate strongly and late. So we tense the muscles in order to slam on the brakes late in the punch.

This “kime point” (focus point) is not the real kime point. The real kime point would be then the weapon hits the target; which would be earlier in the motion. False kime is the tensing of the muscle just before full extension. Real kime is the continued movement through the target (so no locking of the muscles) in an optimum way i.e. biomechanics, motion and mass all converging to give maximum impact.

So I would say that the “tensing on impact” problem – which, as demonstrated, will result in a very weak punch – does not originate primarily with controlled sparring, but instead with a misunderstanding about why we lock the muscles at full extension in kata and kihon.

We should not impact with a straight arm, because that arm has no potential to drive into the target. The arm will be straightening and still moving at impact on any worthwhile punch. The reason we lock the muscles just before full extension (which is NOT the impact point) is for joint protection.

I also think it would be useful to mark the distinction between leaving the arm out and allowing sufficient “time on target”.  At impact, the body should still be moving with the punch driving into the target. Newton’s Third Law will then kick in and decelerate the strike while damage is being done. If I pull the arm away too early – i.e. before Newton’s Third Law has fully decelerated the strike – then I’m taking away from the forward drive and weakening the punch.

By letting the punch run its natural course, and ensuring all the energy from the punch has been transferred into the target, I will hit harder. In this instance, I am not leaving the arm out, or “sticking”, unnecessarily, but in order to ensure the most powerful hit possible.

Kata tends to leave the striking limb “out” so we engrain the idea that the target should slow the strike; not us. This shouldn’t be confused with remaining static at the end of the motion though. It’s “snapshot” that we should instantly move on from.

The instant the punch has come to a natural halt (i.e. been decelerated by the target and not my intervention) then I could pull it away quickly so it can do the next thing it needs to (if anything). In reality, there is a fraction of a second between the return of a punch that is decelerated naturally and one that is pulled; but there is a HUGE difference in power.

A quick flow of high power shots becomes possible when we get used to impacting in that way (target slows the technique, don’t pull any shots, and use the true end of one method – i.e. natural deceleration – as the beginning of the next).

This old video demonstrates the idea. You’ll notice that Joe (young brown belt who is hitting the pads) does “stick” to the pads at impact. You can best see this when it is demonstrated slowly.  He does not snap the punch away, but instead lets the target decelerate the strike. Once it has done so, the hip motion is already primed for the next strike. So what we have here is fast and powerful striking with sufficient time on target as a result.

You also notice, back to the main theme of the thread, that he’s not locking his body at any point. As pointed out earlier, he does not need to because the back shock some say you need to be prepared for – due to Newtons Third Law – is entirely used up in bringing the striking hand to a stop (Newtons First Law). He is moving throughout, and the “stances” (front stance, reverse-cat-stance, reverse-cat-stance) are flowed through … just as Funakoshi, Nakasone, etc said they should be.

Away from power generation, and into the realm of consensual fighting, there can be tactical reasons for leaving an arm out. For example, if I throw a lead hand punch I can deliberately leave it “in the opponent’s face” to obscurer their vision and hide my motion, such that when it is finally pulled away the next technique is only inches away from landing. Such tactical considerations are away from the theme of the tread, but hopefully worth mentioning as a footnote.

Really enjoying this thread!

All the best,

Iain

Kevin73
Kevin73's picture

I think that trying to use physics to teach karate is one of the most misunderstood and misapplied areas I have seen.  I am in complete agreement with Iain's assessment.  Another one that is always misapplied is "F=MA" in regards to punching power.

I dont' think you really need physics to make a student understand ways in which the body can loose power through sloppy joints (elbow, wrist, etc.)

Iain Abernethy
Iain Abernethy's picture

Kevin73 wrote:
I don’t' think you really need physics to make a student understand ways in which the body can lose power through sloppy joints (elbow, wrist, etc.)

I think that’s true because the power of punch is something that can be empirically tested. Feedback from the pads / holder (providing it is held competently and realistically) also makes clear whether a punch was hard or not. We can understand the physics and mechanics empirically; irrespective of whether we can write the formulas out or not.

One thing we can (mis)use “physics” for is justifying false practises if we can sound like we know what we are talking about, and then throw a lot of formulas around to cause confusion. If you want to justify a false understanding of stances as static and locked positions, then talk of Newton’s laws etc may confuse some people enough not question it.

We see pseudo-history (i.e. all the zen, shaolin temple origin nonsense) and pseudo-science (i.e. chi-based striking methods: “fire burns wood”, "this stance give your body a water energy", etc.) used this way too often. It’s hugely damaging to the art I love.

I’m a great believer in following the demonstrable truth historically, theoretically and practically.  We need to be ruthless in ripping karate’s myths away – irrespective of how cherished they may be – if karate is to have a bright future.

Stances are about movement. That’s demonstrable historically, theoretically and  practically. We should not misuse science in an attempt to justify some the inefficient practises karate has acquired.

All the best,

Iain

Mark F
Mark F's picture

I'm sure its possible for physics and maths to describe all of the points raised in these posts in accurate formulas but surely power generation is only a part of the strike.

A rather famous Irish UFC star knocked another competitor out with a rather innocuous punch and stated afterwards "precision beats power and timing beats speed" (I can't be sure if he was the first to say it and don't want to end up mis-quoting a source) 

Iain's last paragraph "stances are about movement" 

This movement allows you to generate the power but with precision and speed. I'm not sure it's wise to isolate them. A maximum force punch thrown without either of the other two elements may have no effect. 

Happy to see the formulas taking those three elements into account :-)

Mark

Iain Abernethy
Iain Abernethy's picture

Mark F wrote:
I'm sure its possible for physics and maths to describe all of the points raised in these posts in accurate formulas but surely power generation is only a part of the strike.

Very true, but it’s the part we are discussing in the thread so we need to isolate it to keep the conversation focused. And while it may be only one part of the strike, I’d argue it is the most important part.

You always need to hit with power. Despite occasional (and easily disprovable) claims to the contrary, people don’t pass out or collapse in pain with taps or touches. Even very weak areas need to be hit with a degree of force. We also have the pragmatic problem of accuracy being difficult to achieve in the chaos of combat.

When it comes to speed we have a similar thing. A fast strike without mass behind it won’t do anything (Newton’s Second Law). Sometimes people try to move the limb so quickly that they don’t get sufficient bodyweight into the strike (arms being quicker and easier to move than the body). When that happens, the fact the strike lands is largely an irrelevance because it achieves nothing. A much used phrase in my dojo is “when given the choice between speed and power, chose power”. A slightly slower punch with mass will do way more damage than a fast one without mass. We always need bodyweight behind the strike.

Mark F wrote:
A rather famous Irish UFC star knocked another competitor out with a rather innocuous punch and stated afterwards "precision beats power and timing beats speed" (I can't be sure if he was the first to say it and don't want to end up mis-quoting a source)

If he means that skill (precision and timing) will beat brute physicality (muscular power and muscular physical speed) then I’d agree. If he is saying power and speed are less important / less effective than precision and timing then I’d disagree. Good technique is what will really generate speed and power. And a p##s poor technique won’t do anything irrespective of how well it is timed or where it lands.

An effective method needs good technique, good timing and good tactics (the what, when and why). You can’t say technique and power (true skilled, technical power) can be beaten if you have timing and precision. You just need to look at Mr McGregor fight to see he’s not lacking in technique or power (which is primarily a product of technique). It’s a nice soundbite, but not one that holds up to close scrutiny in my view.

Mark F wrote:
This movement allows you to generate the power but with precision and speed. I'm not sure it's wise to isolate them. A maximum force punch thrown without either of the other two elements may have no effect.

It is the stances / motion that will also generate the speed. Speed is not just a physical attribute, but primarily a skill that comes from efficient movement. If we can move continually, so that each motion feeds into the next, we can hit very fast and very hard.

I get what you are saying, and I agree that power is just one component; albeit a vitally important one. However, a maximum force punch will only have no effect if it completely misses. High impact applied anywhere to the body will have an effect. There is no part of the human anatomy that is immune to impact. We would also hope that we would be in the ballpark area of where we would ideally like our strike to go too.

It also possible to strike relatively slowly (for a strike) and it still have an effect due to the bodyweight behind it. Indeed, there can be times when it is tactically prudent move a little slower as to not trigger the enemy’s startle response i.e. when delivering pre-emptive strikes.

Peter Consterdine often demonstrates this at seminars by asking people to block some strikes. He throws some quick ones and most of the time they manage it. He then throws a slower one (normally a slap to the face) and the fact it is a little slower means it is not instinctively registered as a threat. They have to think it through, and by the time they done that, they’ve already been slapped … normally accompanied with Peter smiling and saying, “too late!” :-) You can then take it on to the etherfoam and feel firsthand that an increase in mass, and a decrease in speed, can still deliver a shot which will easily put people down.

Mark F wrote:
Happy to see the formulas taking those three elements into account :-)

And this is why we do martial ARTS and not martial science :-) There are tactical factors, human factors, and the fact that real situations are emotional and chaotic affairs that are a long way away from dispassionate laboratory conditions.

It’s also why Funakoshi said:

“Train with heart and soul without worrying too much about theory. Often the man who lacks that essential quality of deadly seriousness will take refuge in theory.”

In summary, I agree that power is only one component of a good technique. However, it is a vital one because a technique without power will always achieve nothing. Therefore we should always be seeking to maximise power. Even if we lose some speed for tactical or technical purposes, or accuracy because of the reality of conflict, a powerful technique will still get things done.

All the best,

Iain

Marc
Marc's picture

Iain Abernethy wrote:

We should not impact with a straight arm, because that arm has no potential to drive into the target. The arm will be straightening and still moving at impact on any worthwhile punch. The reason we lock the muscles just before full extension (which is NOT the impact point) is for joint protection.

I agree. When we practice kata in thin air we have the opportunity to go full extent with full speed. Since there is no impact, we have to deliberately stop the movement at some point. Otherwise we could injure our joints. Another thing that could happen is that by continuing the motion our body balance would be compromised. Apart from looking silly in training, it would also be a problem in an actual fight, if we miss.

Iain Abernethy wrote:

This “kime point” (focus point) is not the real kime point. The real kime point would be when the weapon hits the target; which would be earlier in the motion. False kime is the tensing of the muscle just before full extension. Real kime is the continued movement through the target (so no locking of the muscles) in an optimum way i.e. biomechanics, motion and mass all converging to give maximum impact.

Again, I agree with the idea that the technique actually happens "half-way through". Without motion there can be no impact.

"Kime" is often taught as the tensing of the muscles at the end of a technique. And while its good to stop the motion as we've discussed above, I feel that that does not make good "kime". As far as I understand it, the word "kime" is kind of misused in this way.

The kanji used for "kime" are "決め". It comes from the verb "kimeru" which means "to decide; to choose; to determine; to make up one's mind; to resolve; to set one's heart on; to settle; to arrange; to set; to appoint; to fix". From the meaning of "kimeru" I would suggest that the meaning of "kime" as a noun would be something like "decisiveness" in our context. A determination to make this next technique the one that will end the fight. Think "him down now" or "mentality over technique".

So kime, as I understand it, is the mindset behind the technique. Otherwise the technique might come out half-heartedly. It might still be precise and well executed. But it would lack spirit.

Sometimes that's what I think is missing in modern kata competition performances. They are incredibly accurate, they show speed, stability and timing. They are very focused on each technique. You can even hear this amazing "snap!" sound. Great kime? Well, when its too polished I feel that something is missing. And I think what's missing is that fight-ending mindset - kime.

Kime is not a way to perform a technique. It is the decisiveness with which the technique is initiated. This then reflects in the way a technique is executed.

All the best,

Marc  

Stevenson
Stevenson's picture

Edit:

Hi Iain,

I get and agree with the sentiment beneath this, but this is not to do with "structure" but with the efficient generation of power. I can shift my weight and rotate my hips, but if the arm motion does goes provide an efficient way of transferring the bodyweight shift, then it will be a weak punch.

Well I think it is about structure - it's about the point of delivery rather than the point at which it is generated. If your arm collapses at the wrist, elbow, and shoulder it will be less effective than it would have been if the arm's structure had been good don't you think? I have in mind 2 different kinds of punch:

1) A power punch. By this the punch is totally dynamic; full hip hip rotation, body weight transference and fully committed punch.

2) A pushing punch, this is where not just the arm but specific muscles lock out at the exact moment of impact in a snapping motion. This is more "grounded" in nature than fully dynamic punch. (The breathing here (for me) is important - Taira Masaji describes as a "sneeze" - really explosive expulsion of energy and then instant relaxation. 

Either way, what you trying to do is direct as much of your energy into the target as you can and not lose it through your own structure, whether you build up momentum (the power punch) or Impulse (the pushing punch). In the former you use body weight and momentum to slow down the decceleration, but in the latter, you use bio-mechanics ie good "strucutre". The advantage with the power punch is obviously power, but the "pushing" punch has the virtue of being able to be delivered faster and so without telegraphing of movement, and from a smaller range, and without over committing. The structure matters differently depending on body type, but surely you've seen the slighter frame types of people with their shoulders popping back, knees collapsed in, delivering a pretty ineffectual strike? 

I think we are getting way too caught up on Newtons third law. The physics of a punch are really complicated, force is just an element. Don't you think it might be better conceptually to appeal to Newtons second law; conservation of energy? (Edit: I reread the thread and missed an earlier post where Iain makes the same point...)

When striking you are delivering ENERGY to a target. That energy can be instanteous or over time. It can be used to strike or unbalance and its effect is dependant on the surface area and elastic properties of the target and the delivery mechanism. So if we consider what we need to do to ensure the energy is focussed correctly depending on the result we are looking for and thinking about where energy might be lost in our own structure and technique we can make them more efficient.

We can incorrectly think of “stances” as being static instead of snapshots of motion i.e. places we move to and through. Standing in a static position (“a solid stance”) in front of a totally static target like the makiwara, which will send a good deal of force back into the body due to how unrealistically solid and fixed it is, gives a way of striking that is antithetical to real world combat and impact. It needs to be suplimented with more realistic impact equipment.

I completely agree that there is a danger we view "stances" as being static, but I think the value of something like makiwara is to explore where in your structure energy is being distributed. Ensuring you engage good structure means at the point of impact you are in the most efficient strucutre possible will improve your strike or technique. Edit: A further point regarding "locking-out"; the "good" structure we aim for at the precise moment of impact is momentary. But that last inch/two inches of a punch from the point of impact to full extension is the bit we want to look at, and if we engage the correct muscle groups we get a delivery mechanism that doesn't lose energy we want directed into the target. Just look at inexperienced students punching a focus pad. If their wrist is collapsing, the elbow pointing out and the shoulder popping back, nice weight transference is being wasted!

This is just my rather unworthy view, but to me each stance is a structure that has a purpose. Zenkusu dachi is for forward or backward pressure. Either pushing a car or holding up a toppling wardrobe. Directing energy forward or resisting coming back. Shika dachi is for directing energy down or in preparation for driving up and so etc etc. So rather than think of a structure as being "static" I like to think of them as solving a problem I am presented with, and by practising them I can get that problem solved more efficiently. And I like to think of pitting my "structure" against an opponents strength - that bones are stronger than muscles.

I guess "alignment" works if "structure" implies something static - I can see how that might be. But I am not thinking of structure as being static, I am thinking of it being the most energy efficient use of my body for the job I'm trying to do at the time. (God knows I need to be energy efficient). Edit: I am envisaging structure as one point in time to another - if I am trying to use shika dachi to achieve something, but my core isn't engaged, my knees aren't pulled out, my weight isn't centred, them my structure is poor and the technique I am trying to perform is likely to be less effective or possibly not work. That's "structure" to me and I'm not sure "alignment" would be the right word in that context. I'm also thinking of my favourite strike, the drop step lunging oi tsuki - that is all about structure. I practise it trying to feel how by bones align, from fist, through my shoulders, all the way down to my back heel driving into the ground. I can land the strike a bit too early and it'll still be effective since I am using it to upset an opponents forward momentum.

------

While looking for the article I was talking about I came across this:

http://journals.sagepub.com/doi/pdf/10.2466/pms.109.1.295-303

I haven't gone through the article above in much detail but they seem to conclude that reaction time and force was greater for palm strikes which they suspected was because of energy loss in the wrist during punches. Precision was lower for palm strikes. Take a look - it's interesting - though it's a very very small sample - I wouldn't draw any firm conclusions from it.

This is the article I spoke about earlier: J.K. Gulledge and J. Dapena in Journal of Sports Sciences 2008 

https://www.ncbi.nlm.nih.gov/pubmed/17943591

That one examines reverse punch and 3-inch punch. Not necessarily germaine to the discussion but interesting none-the-less.

Stevenson
Stevenson's picture

I have another (hopefully clarifying) thought:

You always need to hit with power. Despite occasional (and easily disprovable) claims to the contrary, people don’t pass out or collapse in pain with taps or touches. Even very weak areas need to be hit with a degree of force. We also have the pragmatic problem of accuracy being difficult to achieve in the chaos of combat.

In response to MarkF's post Iain gets to the heart of an interesting question. I don't think anyone proposes that power is not important. But can't we ask "how much power"?

In karate (and much in life) there are trade offs. To get absolute maximum power means a huge amount of body movement and transference of body weight. But do we need to hit that hard?

I'd like to argue that maybe we don't. I know guys whose half power punches would more than get the job done I personally suspect. So if we trade in some power for precision or speed wouldn't that potentially make their strike more effective? Ie, you won't punch as hard but you are more likely to hit your target and more quickly.

I also think body type comes in to play a bit. The bigger and more powerful you are in general the slower perhaps. So you have power to spare in favour of speed and accuracy. The smaller and lighter you probably should ensure you maximise power. Is this fair enough?

BTW - really enjoyed this thread! It's been one of the more stimulating discussions on this for a while.

Iain Abernethy
Iain Abernethy's picture

Stevenson wrote:
Well I think it is about structure - it's about the point of delivery rather than the point at which it is generated. If your arm collapses at the wrist, elbow, and shoulder it will be less effective than it would have been if the arm's structure had been good don't you think?

I’d still go with “alignment”. The elbow and shoulder joints will be moving on contact so the engineer in me views this as something mechanical, as opposed to structural. Again, this is because I think "structure" infers something static; whereas "mechanical" infers motion.

A punch is the body moving and therefore mechanical terminology would seem most appropriate. We are also essentially talking about “biomechanics” and hence “mechanics” / “mechanical” terms would again seem to be the natural lower tier terminology. 

Good mechanics (including body mechanics) requires the correct alignment of all the moving parts. To my mind, the alternative phrase, “the correct structure of all moving parts” points to the nature of the parts themselves, and not the way they are interacting.

I think that “structure” is a term that can cause confusion, obscures the kinetic chain of efficient striking, infers something static, and that potentially opens the door to talk of needing to “lock” the body as a solid “structure” on impact (as we’ve been discussing); all of which is antithetical to real world power striking.  

Of course, people can describe it how they like but the proof of the pudding is how hard they hit, and, if they are teachers, how hard their students hit.

I can see the “structural points” you make being more relevant to some parts of grappling (ability to withstand force and maintain stability). I’d be totally happy to use the word “structure” in that context. However, when it comes to striking – the theme of this thread – I’m firmly of the view it is mechanical (specifically biomechanical), not structural, and that “alignment” would therefore be a better word to reflect what is happening. However, to each his own. "A rose by any other name would smell as sweet" and the true test is how sweet the rose smells / how hard we hit.

Stevenson wrote:
But can't we ask "how much power"?

As much as possible. The stresses of conflict mean we need plenty of slack in the system. I was once privy to a series of conversations with some serious guys (all of whom were skilled martial artists) who have faced serious violence at serious levels. There was much talk about how the beautiful bag folding punch you had in the gym greatly deteriorated under stress. Me being me, I tried to get them all to independently out a figure on that deterioration. Most went for around a 80% loss; which I think is fair. So, for it to be relied upon, we want a punch that is at least five times more powerful than is needed to cause a knockout. Essentially, we should never compromise on power … just as we should never compromise on technique, timing, skill, etc. We should always seek the most we can get.

One of my teachers told me a story of when he was once working on the door and he was trying to eject a guy. The guy was causing trouble and had his back to the main doorway. The guy was getting agitated, but was a little worse for wear. My sensei, being a nice guy, figured a moderate front kick was required to take a little of the fight out of him, and to help get him through the door so it could be closed after him. My sensei throws the kick, which hits the guy and folds him over. However, because it was a moderate kick, it did not finish the guy and he was able drive the glass that was in his hand into the face of my sensei. The glass broke and he feels a warm liquid flowing all over his face (assumed to be blood). Things step up a gear and the trouble maker is quickly sorted out. My sensei puts his hand to his face and it is covered in blood. He assumes his face has been utterly shredded. He goes to the bathroom and finds the cuts were relatively minor and the warm liquid was a mainly booze (obviously been in the hand of the trouble maker for a long time) which had diluted the blood and made it seem worse than it was. A close call though. The lesson he learnt was never to hit people softly. You either hit them or you don’t. Do all you can to avoid needing to hit people; but if you can’t then you need to do it right.

Away from self-protection into pure martial arts; we should want the best possible technique, and the best possible technique will have the most possible power.

Stevenson wrote:
To get absolute maximum power means a huge amount of body movement and transference of body weight.

These things always need balanced against tactical considerations. However, I don’t think it’s true that “huge” movements are needed. Efficient motion can generate masses of power.

Stevenson wrote:
So if we trade in some power for precision or speed wouldn't that potentially make their strike more effective?

I don’t think there’s a trade to be made there. Why is a powerful technique potentially less accurate than a weaker one? I don’t see the link. It is possible to have power and precision and I can’t see why the two have to be mutually exclusive. Same with speed. In my experience, speed is a skill and the motions that give us maximum functional acceleration, and the highest rate of fire, are also the ones that give maximum power.

As per the above post, we can have meaningless speed (a fast “arm only punch” that does nothing), and we need power; irrespective of precession. We also need to remember that, in reality, getting that precision is not as easy as some think.

I can’t see a viable trade-off for deliberately seeking less power. That makes no sense to me. We need power, and the more the better.

Stevenson wrote:
I also think body type comes in to play a bit. The bigger and more powerful you are in general the slower perhaps. So you have power to spare in favour of speed and accuracy. The smaller and lighter you probably should ensure you maximise power. Is this fair enough?

I don’t think so. I immediately think of someone like Brian Seabright (head of the BKA) who is a small guy but incredibly powerful when his hits. This is because his technique is very good and hence his bodyweight is behind every strike he throws. You can see Joe (the brown belt in the video above) and he’s not a big man (at least he wasn’t when that was filmed) and yet he still has massive power. I also think of Jenny, who is a small black belt who has trained with me since childhood, and her striking power hurts my hands through the pads. At the other end, I think of Peter Lakin (another of the people I train with regularly) who is a big guy but is incredibly quick. Again, this is mainly down to technique and effective transitions. I’m also of above average build (16 stone), but no one has ever accused me of being slow :-)

Having extra bodyweight (i.e. being bigger) obviously gives you a greater potential to hit harder, because you have more weight to put behind the technique. However, without the refined technique to do that it will still be a weak punch. Some people are physically quicker than others. However, the whole point of martial arts is not to “make do” with what we have genetically.

Power and speed are primarily a result of good technique. Therefore, what we should find is that the hardest hitting and fastest students are the ones with the best technique. If we have a big beginner hitting harder than a smaller kyu grade (let alone a dan grade), then whatever we are teaching is obviously serving no purpose. That’s not how it should be.

Everyone, irrespective of build, should seek the highest level of technique possible. And that will mean they will all seek to hit as hard as possible (and as fast as possible, and as accurately as possible).

I’d be very concerned if anyone was telling small people that the will never have meaningful power, or that big people will never be quick. Good technique and good instruction makes both those statements untrue.

Stevenson wrote:
BTW - really enjoyed this thread! It's been one of the more stimulating discussions on this for a while.

Yeah, me too! I love the ones that get in-depth, but have a variety of viewpoints expressed. I think they make for the most interesting reads and the web statistics largely reflect that too.

All the best,

Iain

Stevenson
Stevenson's picture

 I’m also of above average build (16 stone), but no one has ever accused me of being slow :-)

Absolutely not! Man you look like you have your own spacetime reality going on when you move! Like someone has sped up the film...

But, I also wonder if that colours your point of view.....A question I have been meaning to put to you for a while is to do with dealing with different physiologies. I find it amazing how quickly and relaxed you move especially for your size (....which of course comely and fashionable...ahem...). Some of the bunkai drills you teach and that we practise feel awkward to me because I can't make a repeated move without tensing up, and that slows me down. It's not just in karate either, as a musician this was always a problem and to improve my technique there and in karate, I have to focus 100% on relaxation, on making sure I don't have muscles acting against one another. In Chinese medicinal terms, ensuring my "chi" doesn't become "blocked".

So I find, for example, repeated actions cause tension to build quickly. I much prefer moving immediately on to a different technique using a completely different set of muscles, especially using anything returning from a strike or some technique to be involved in the next during the return. I can launch a technique nice and loose and with plenty of power and speed and explosiveness, what I can't do, like you and many others seem to be able to do easily, is return completely to relaxation right away - it just doesn't come naturally. I have breathing tricks and vocalisations that help, although I feel like I must sound like a right tit.

Back to the point of with trade a little power for speed and accuracy, I don't feel convinced yet. Trying to see it from your point of view it sounds to me like you are worried about intent, would that be right? So if you think you are backing off power it will affect intent. The examples you posted seem to me about that, for example your sensei who used a moderate powered kick because that's all he thought he needed, but it isn't really what I mean.

I am thinking about training for speed and precision, especially if you are already capable of a fair bit of power. It doesn't mean you don't hit with ANY power, nor INTEND to hit less hard, you still hit as hard as you can, but prioritise speed and accuracy. I'm thinking of the heavier person in particular who does NOT move terribly fast. To hit absolutely as hard as they possibly can and train for it risks (to my mind) movements that telegraph intentions. They don't need to put all their body weight into a strike because they are sufficiently heavy they will have enough inertia for it to be as effective as a full powered strike from a lighter person. But that inertia also means they don't necessarily move around as quickly or with as much agility.

I just can't help the feeling that different body types means different priorities. If the student was smaller and lighter and therefore faster then power becomes the priority, and the heavier student that doesn't move so well can shorten their technique and focus on sped and accuracy.

I’d be very concerned if anyone was telling small people that the will never have meaningful power, or that big people will never be quick. Good technique and good instruction makes both those statements untrue.

 Well I would never tell a smaller person that. An illustration was once made to me that if you thought 50 kilos couldn't hurt you, lie on the ground holding a 50 kg weight above your face and then drop it. On the other hand, size and strength does matter in a fight. That's why they have weight divisions in professional fighting. I would always caution a skillful 50kg woman not over esitmate her chances against a 120 kg gorilla with no skill.

But really what I am suggesting is completely in accordance with your statement here. I am saying exactly that small people should have power and bigger people should be quick. What I am suggesting is, there may have to be trade offs to accomplish that. A small guy who weighs 60kg can never get more than 60 Kg of body weight behind his strike, but obviously a guy weighing 100 kg can. I'm not suggesting that any technique, even if some power was sacrificed for speed, should ever not have full focus though.

Iain Abernethy
Iain Abernethy's picture

Stevenson wrote:
I find it amazing how quickly and relaxed you move especially for your size (....which of course comely and fashionable...ahem...).

LOL. I’ll take that :-)

I promise my speed comes from technique. It’s not a physical gift I’ve been blessed with. Anyone can move quickly if they are taught to move efficiently and transfer from one method to the next efficiently. I was taught it, and I teach others it.

Stevenson wrote:
Some of the bunkai drills you teach and that we practise feel awkward to me because I can't make a repeated move without tensing up, and that slows me down …

So I find, for example, repeated actions cause tension to build quickly … I can launch a technique nice and loose and with plenty of power and speed and explosiveness, what I can't do, like you and many others seem to be able to do easily, is return completely to relaxation right away - it just doesn't come naturally.

To me, this sounds like you need more practise so you can move without tensing at the wrong places. What you are describing is tensing (or “locking”) unnecessarily. Lots of karateka have this problem and it, in large part, comes from what we’ve been covering in the thread i.e. a misunderstanding why the muscles tense at the end of a technique in line work, thinking structurally and not mechanically, seeing stances as fixed position and not places we flow to and through, believing the makiawara to be an accurate representation of real world hitting, and so on. That’s why I’ve had a lot to say in this thread. It does not have to be like that. All of these problems can be avoided and karate will be better for it.

Stevenson wrote:
Trying to see it from your point of view it sounds to me like you are worried about intent, would that be right?

No. I am taking specifically about power.

Stevenson wrote:
The examples you posted seem to me about that, for example your sensei who used a moderate powered kick because that's all he thought he needed …

He should have hit him with more power.

You asked how much power do we need, and my thoughts are, as much as we can possibly get.  

Stevenson wrote:
I'm thinking of the heavier person in particular who does NOT move terribly fast.

As per the above post, I would see that as a technical problem and I see no reason why big people can’t move quickly. It’s fixable and does not need to be accepted as an inescapable given.

Stevenson wrote:
To hit absolutely as hard as they possibly can and train for it risks (to my mind) movements that telegraph intentions.

That would be the tactical considerations I mentioned. It’s a mistake to think that a powerful blow has to be clumsy, big and inelegant. Good technique will still generate massive power without excessive motion that could slow the rate of fire or excessively telegraph. You can train for maximum power within a sound tactical framework.

Stevenson wrote:
They don't need to put all their body weight into a strike because they are sufficiently heavy they will have enough inertia for it to be as effective as a full powered strike from a lighter person. But that inertia also means they don't necessarily move around as quickly or with as much agility.

Everyone, big or small, should always train to hit as hard as possible.

The inertia takes more energy to overcome, so a big person will burn more energy when moving than a smaller one will (a stamina issue in the longer term), but I know plenty of big, quick and agile people. Again, I would see that as technical issue if the big person was moving without agility.

Stevenson wrote:
If the student was smaller and lighter and therefore faster then power becomes the priority, and the heavier student that doesn't move so well can shorten their technique and focus on speed and accuracy.

I’d refer to the previous post for my take on this. I don’t think the assumptions being made about body types or what a powerful technique requires is correct. It’s certainly not been my experience.

Body types do coming to play of course (specifically with regards to choice of methods, tactics, preferred throws, etc.), but for striking we all want powerful punches and the mechanics for achieving that remains constant.

Stevenson wrote:
On the other hand, size and strength does matter in a fight. That's why they have weight divisions in professional fighting.

Absolutely. But that’s beside the point. When it comes to striking the technique does not change. The mechanics of boxing  are the same throughout all the weight categories. Heavyweights and lightweights throw their punches in the same way. In this thread we are talking about the physics and biomechanics of punching. That is universal.

Stevenson wrote:
A small guy who weighs 60kg can never get more than 60 Kg of body weight behind his strike, but obviously a guy weighing 100 kg can.

I get what you are saying with regards to mass.  As I said above, a big person has more mass to put behind his punch … but there is a mechanically optimum way to punch which will get maximum weight behind the technique; irrespective of what that bodyweight is. That optimum method is what all martial artists, big or small, should be pursuing.

All the best,

Iain

Stevenson
Stevenson's picture

Well this discussion has thrown up some interesting issues, but if you'll forgive my making another few comments because I still think we mgiht be talking past each other a wee bit, but ther are also a couple of points I disagree with and I'd like to chuck them in there for the fun of it.....

I promise my speed comes from technique. It’s not a physical gift I’ve been blessed with. 

I am sure it comes from technique AND phsical gifts you have been blessed with. If you took someone with Muscular Dystrophy with your size and weight I'm not sure you will get such good results no matter how much they practise technique. I realize that is exageration for effect but I'll come more on to this.... 

To me, this sounds like you need more practise so you can move without tensing at the wrong places. What you are describing is tensing (or “locking”) unnecessarily. Lots of karateka have this problem and it, in large part, comes from what we’ve been covering in the thread i.e. a misunderstanding why the muscles tense at the end of a technique in line work, thinking structurally and not mechanically, seeing stances as fixed position and not places we flow to and through....etc

No this is not at all what I am talking about. I appreciate you have a problem with the word "structure" because it suggests something fixed and static. But for me (and possibly others), it is simply the shape and form that has to function effeciently. If you do not have good structure, fantastic weight transference or dynamic movement won't help you, because you won't be able to put on the road, like a V8 engine powering bicycle wheels. You might have all the power in the world but no way to deliver it. For some people it really matters that they think about this. However limited my experience is, it is a universal truth I have found; in MA, sport, or any kind of physical pursuit, some people have better natural postures and structures than others, around which they can base their movements, moving their bodies, so that at the point of delivery, a technique is in a sound structure to accomplish the task at hand. Fingers playing a piano are continually moving but their structure has to be right, curved just so, the arms and wrists have to be aligned so that the muscles and tendons don't meet any resistance, and the upper body posture relaxed but firm enough to support quick lateral movements. Some people just do it naturally, others have to learn it.

In my specific case, and some others I have seen too to greater or lesser extent, my problem with tension I suspect is to do with muscle elasticity - the ability of muscles to return to normal after a contraction. My problem is not tension on point of impact, it's repeated movements. Or it could extensibility, muscles returning to normal after being stretched. Whatever, I can get around slowness by ensuring I don't repeat an action, and of course training to be explosive and focussing very hard on eleminating tension. I have exactly the same problems playing musical instruments - it's just not something that comes to me naturally.

I do marvel at your fuidity of movement though...it's very impressive.

As per the above post, I would see that as a technical problem and I see no reason why big people can’t move quickly. It’s fixable and does not need to be accepted as an inescapable given.

And this throws up one of the great questions I ponder about martial arts - especially practical martial arts. Do we work with what we have been given and accept it , or do we try to overcome it? If you are an old man with dodgy hips and knees, I don't think really dynamic motion is going to be easily attainable. But does that mean you can't hit with power? I'd argue, no it doesn't, but it might have to be found in another way. And while I don't think it means we should give on trying to improve our flexibility and range movement,t it can be hard to know how far along the road of diminishing returns to travel.

I do see some technical limitations to speed and power due to size and body type. To illustrate this I looked at the world records in olympic weight lifting. This gives us emperical comparison of size to strength and while not perfect is probably a fair. In each weight division we can reasonably say that we have around the limit of what is capable for that size of person. We can then work out a ratio for strength to weight (as percentage of body weight). It's interesting! I hope you think so too. Each of these is for male clean and jerk:

56kg       170kg     303%

62kg       177kg     285%

69kg      196kg      284%

77kg       214kg     277%

85kg      217kg      255%

94kg      224kg     238%

105kg    237kg    225%

105+(162kg) 263kg   163%

For every 8kg division higher you lose on average 13% of strength to body weight.

I put it to you that this ratio will be reflected in other forms of use of power. So simply moving around, accelerating your body, moving your arms etc will diminish the heavier you get as a consequence of the square cube law.

That's it for now - please do let me know (gently) if I am flogging a dead horse or teaching grandmother to suck eggs or whatver. It's a genuinely interesting conversation to me.

Iain Abernethy
Iain Abernethy's picture

Hi Stevenson,

Stevenson wrote:
I am sure it comes from technique AND physical gifts you have been blessed with.

My family and early school reports will confirm I am far from being athletically gifted :-) What I have comes from training. I also know that I have all kinds of shapes and sizes in my dojo and all can hit hard and quick with sufficient practise. I think people are too quick to use the fact that they are not genetically gifted to limit the levels they can reach.

At very high levels (which I’ve never reached) genetics plays a part in providing the fine line between the good and the truly great. However, as is the theme of this thread, I maintain the optimum methods for developing power apply to all people. The methods by which individuals will achieve their maximum power are determined by universal physics and common biology.

Stevenson wrote:
You might have all the power in the world but no way to deliver it.

I can’t follow this. Power is something enacted when it comes to striking. You can’t have it and not deliver it, because delivering it is it.

Are you conflating powerful impact with physical strength? They are not the same (as per above posts).

Power is a technical thing. A learnt skill. It’s not something we have. It’s a result of something we do.

Because it appears when we “do” it can’t be something we have. It’s little like saying I “have” a jump, or a run, or a throw. We don’t have those things, we do those things. They only appear when we do (or deliver) them.

As I say, I can’t follow that line.

Even someone not genetically gifted can hit powerfully with good technique. Maybe not as powerfully as a genetically gifted, highly trained world-class athlete, but powerfully nonetheless.

Your chart about strength to weight ratios is not really relevant because we are talking about the physics and techniques of striking in this thread … but to run with it, we can observe that all lifters use the same techniques irrespective of size. An optimum way of lifting has been identified, and both lightweights and heavyweights lift in the same way … and that’s because it is physically and biomechanically most efficient. They don’t have a “heavyweight technique” and “lightweight technique” for lifting. They lift the same way … and striking is the same (the point I'm making)… as per the boxing analogy I made above.

I think we are staring to muddy the waters and get wide of the point? This thread is about the physics of striking and how we can develop the most powerful strikes possible. Everyone who wants to hit hard should aspire to the same optimum methods. Body type was raised and it was suggested that some can’t be powerful or fast, or that different methods should be used. My experience tells me this is not true (provided they are able to practise martial arts in the first place) and that the laws of physics and what constitutes a powerful technique applies universally to all martial artists.

All the best,

Iain

Stevenson
Stevenson's picture

I can’t follow this. Power is something enacted when it comes to striking. You can’t have it and not deliver it, because delivering it is it.

Are you conflating powerful impact with physical strength? They are not the same (as per above posts).

 No I am not conflating power with physical strength, I am making a distinction between power generation and power delivery. If a car has a powerful engine, but a gearbox designed for a less powerful car, the engine will blow the gear box. It's possible to develop power, but not deliver it effectively. In martial arts terms that means you need good structure at the point of delivery of your technique, which up to that point was largely about power generation.

In Goju, we have a kata designed specifically to train for this purpose - Sanchin. It's not a kata that has bunkai per se, it a study of structure. When we do it, we are supposed to focus heavily on making sure we have tension where it is needed for a strong structure and relaxation for anything that is not needed. We often refer back to sanchin when looking to improve our technique for any given application. They say, "don't look for bunkai in your Sanchin, look for Sanchin in your bunkai". It's incredibly effective - understanding the structure of Sanchin and relating it to what you are trying to do can illict instant imrpovements.

Because it appears when we “do” it can’t be something we have. It’s little like saying I “have” a jump, or a run, or a throw. We don’t have those things, we do those things. They only appear when we do (or deliver) them.

 Sure! But that's not the argument. Some people can jump high naturally as a result of favourable biomechanics. Some people can run really fast, and with training really, really fast, because of faourable biomechanics, and or because of more fast-twitch muscles than usual. I used to play Australian Rules football. A footy team requires a range of player types, and they all have strengths and weaknesses. Tall players hold down central positions along the spine of the ground, strategically placed where they can meet the ball at its highest point. The smallest players are generally the ones who follow the ball around, because once the ball hits the ground they have the advantage being faster, and more agile. They can also run faster for longer too, because of their size to strength ratio.

All players, regardless of size must train for speed - though some more than others. All players must train for strength or endurance - but again some need it more than others because of their strategic role in a game. Everyone trains for each element required, but they focus on what is strategically important.

They don’t have a “heavyweight technique” and “lightweight technique” for lifting. They lift the same way … and striking is the same (the point I'm making)… as per the boxing analogy I made above.

 Sure, but they are simply trying to generate as much power as possible because that is the goal, when striking someone the goal is to knock them out. I can't help the feeling that for a smaller person power should be a greater priority than for a bigger person who might be slower and therefore train for speed - much as you have. And I also can't help the feeling and experience that the exact same technique will be slower for a larger person than for a smaller, leading me to think about the question of prioritising speed for power - and by speed I mean the trigger point to impact.

I suppose what is really in my mind is the question of telegraphing, and really making sure that your hit lands. Which would you choose, a greater chance your strike will miss but for a greater chance your strike will get a knockout, or a greater chance your strike will hit, but the possibility it might not get a knockout? (not a leading question - I'm asking sincerely).

And I must stress, I am not saying that we take anything off the strike - I am not saying it should be half hearted - I guess I am saying I would rather make sure I actually hit (speed) even if it means sacrificing my most powerful strike for a faster one (in terms of time from trigger).

So there are 2 things we have been debating 1) is structure and whether it is useful to think of power generation in those terms and 2) the degree to which we take individual biomechanics, size and weight into consideration in relation to our strategic need to hit someone quickly, and effectively.

Once again thank you very much for you time and patience discussing this with me. Such cut and thrusts can be incredibly instructive. I'll be training much more thoughtfully....

Iain Abernethy
Iain Abernethy's picture

Hi Stevenson,

Stevenson wrote:
Once again thank you very much for you time and patience discussing this with me. Such cut and thrusts can be incredibly instructive. I'll be training much more thoughtfully....

I totally agree and thanks for adding to what I feel is a very interesting read.

I think we are now starting to repeat things already said though. Things raised in your last post like alignment vs structure, potential telegraphing, physical attributes vs technical attributes, power vs. speed etc have been raised in previous posts. My views on them are also available to be read in previous posts. I’d therefore refer readers back to those posts in order to avoid restating what has already been said.

For example, in the last post you wrote:

Stevenson wrote:
I suppose what is really in my mind is the question of telegraphing, and really making sure that your hit lands. Which would you choose, a greater chance your strike will miss but for a greater chance your strike will get a knockout, or a greater chance your strike will hit, but the possibility it might not get a knockout? (not a leading question - I'm asking sincerely).

We’ve discussed all elements of that question previously.

A few days ago you wrote:

Stevenson wrote:
To hit absolutely as hard as they possibly can and train for it risks (to my mind) movements that telegraph intentions.

To which I replied:

Iain Abernethy wrote:
That would be the tactical considerations I mentioned. It’s a mistake to think that a powerful blow has to be clumsy, big and inelegant. Good technique will still generate massive power without excessive motion that could slow the rate of fire or excessively telegraph. You can train for maximum power within a sound tactical framework.

In another previous post you said:

Stevenson wrote:
To get absolute maximum power means a huge amount of body movement and transference of body weight.

To which I replied:

Iain Abernethy wrote:
These things always need balanced against tactical considerations. However, I don’t think it’s true that “huge” movements are needed. Efficient motion can generate masses of power.

You also wrote:

Stevenson wrote:
So if we trade in some power for precision or speed wouldn't that potentially make their strike more effective?

To which I replied:

Iain Abernethy wrote:
I don’t think there’s a trade to be made there. Why is a powerful technique potentially less accurate than a weaker one? I don’t see the link. It is possible to have power and precision and I can’t see why the two have to be mutually exclusive. Same with speed. In my experience, speed is a skill and the motions that give us maximum functional acceleration, and the highest rate of fire, are also the ones that give maximum power.

As so on.

I am therefore just be repeating myself when I say that power does not require excessive motion (we need efficent motion). I also don’t see how power and precision are mutually exclusive. I also don't see striking power as a physical attribute that one inherently possesses. How hard people hit will be overwhelmingly determined by technique.

As per the previous posts, I don’t accept the assertions made. We've been over this before though.

It’s a discussion we’ve had.

One point where I do feel you have misrepresented what I said in your last post:

Stevenson wrote:
I can't help the feeling that for a smaller person power should be a greater priority than for a bigger person who might be slower and therefore train for speed - much as you have.

Again, my views on this are discussed above. It is not correct to say, “much as you have”. I've done no such thing.

As I hope the posts make very clear, I don’t share this assumption. I therefore have not based my training (or teaching) on what I believe to be a false assumption. I am not a “big person who has trained for speed” as has been suggested. That would be antithetical to the view I have expressed in this thread.

I am believer in everyone (big or small) seeking optimum striking technique (which will give optimum power and speed). I have therefore definitely not sought to train for speed over power (as you suggested) based on my build. You are of the view we should do that, I don’t share that view. But you’ve superimposed it on to me in the above post.

I don’t believe in the premise and I think I’ve been clear in stating the exact opposite. I have trained for good technqiue and that has given me power and speed; just as it does anyone, irrespective of build. Good technique will enable anyone to hit with maximum power. Speed and power are primarily technical; and good technique should be sought by all. If a big person were to seek speed, separate from power, then they are seeking poor technique. Not something I would advise or do. I'll refer people to the above posts rather than restate this in full again.

Stevenson wrote:
So there are 2 things we have been debating 1) is structure and whether it is useful to think of power generation in those terms and 2) the degree to which we take individual biomechanics, size and weight into consideration

I’d agree, but I feel we’ve now done that in more than sufficient depth. We are now in danger of endlessly repeating our own views at one another. It’s clear we don’t agree, and that’s fine. To my mind, the aim of all debates here is not to win converts, but to ensure the topic is thoroughly explored and hence to provide a good resource for members and readers. I think we’ve done that.

I think we have both clearly expressed our views, and explained why the other’s viewpoint is not one we can agree with. We are therefore both quite happy with the positions we personally hold, and readers should be able to appreciate where we are both coming from and decide what does not and does not chime with their own understanding and experience. I’m happy to explore any previously undiscussed angles, but I’m not sure there are any on this specific topic. Persoanlly, I think that’s job done :-)

Thanks very much for the great contribution to what has been a very in-depth thread.

All the best,

Iain

Iain Abernethy
Iain Abernethy's picture

I thought a video may help round off the topic and give an illustration of what I mean. Words alone can often be confusing because they have different connotations to different people.

Here is video (taken last night) of Callum. He’s 14 years old, ranked 5th kyu, and not physically big. I chose him because it shows what is possible through technique; even from those who have a relatively modest amount of training (i.e. he’s not a dan grade with decades of practise).

Callum gets off four full power shots in around a second. That’s something everyone should be able to do as a baseline standard. They just need to be taught how.

While the mic on my phone didn’t really do it justice, there was one hell of a BANG on all four shots landing. Tim (holder) is a considerably bigger than Callum and has, quite righty, locked his arms while holding. If you watch Tim’s head, you can see Callum is moving Tim’s entire body on every shot. That’s some degree of impact. It’s not muscle or weight, but technique that’s giving him the power.

You’ll also notice how Callum is moving fluidly and that his motions are not large or extraneous. He’s also not “locking” his body to deal with the backshock at any point (because it’s not needed). His slap is getting dropped on the preparation (we can work on that) but, even with that, he is still landing four full power shots – all of which move the much larger individual in front of him trough the pads and arms – in around one second.

That rate of fire combined with the tight motion overall gives the enemy little time (the average visual reaction speed for humans is 0.25 seconds) to react or cues to act on. He’s also consistently hitting the pad with a good degree of accuracy.

No compromise on speed, power or accuracy.

Power   ✓

Bodyweight in all four shots.

Speed   ✓

All four shots delivered in 1 second.

Accuracy ✓

All four strikes squarely hit the target.

Tactically Sound ✓

Extraneous motion avoided, rate of fire likely to overwhelm (exceeding average human reaction speed), relaxed throughout, and balance and posture allows continued delivery of strikes.

His hips are leading his hands so there’s good torque and the shots are “overlapping” which permits speed. If you watch the hip movement (slowed down) you can see how the hips are firing the first shot forward (so it’s not a weak “arm only” strike). Also, notice how the hips then don’t stop moving throughout the entire sequence. No locking or tensing. None of the strikes are from a “dead start”; which is what helps gives the speed. The rotation of the body also gets weight behind each shot and that gives the power. Speed and power are primarily technical (not physical). They also support one another. What is giving the technique power is also adding to the speed.

I’ll reemphasise that this is a 14-year-old, 5th kyu, of slight stature. That’s why this video is a good illustration of concept. If we have a situation where dan grades can’t match a 14-year-old, 5th kyu for speed and power (four full power shots in a second), then we have a problem in with what the dan grade is being taught. Four full power shots in a second is baseline stuff. It’s not the sole reserve of the elite or the genetically gifted. Everyone can do this.

The speed and the power come from the technique. Callum was taught to do this. Just as I was. Just as everyone in the dojo, big or small, is taught the same method. And we know from long experience that it works for all.

It’s a method that gets maximum power and speed for everyone. We don’t teach big people one method and small people another when it comes to power striking. It is also my firm view that we don’t need to pick if we want power, speed, accuracy or efficient motion. We can have them all because they are not mutually exclusive.  Indeed, good technique demands we have them all.

I hope the video helps show that power, speed, accuracy and efficient motion are all part of what constitutes good technique. I also hope the video helps show that absolutely everyone should be able to get four or more full power strikes off in under a second. It’s not the sole reserve of the elite level, nor are speed and power God-given attributes. Anyone can hit hard and fast. It is all down to learning what are relatively simple principles.

All the best,

Iain

Stevenson
Stevenson's picture

Hi Iain,

Thanks very much for the great contribution to what has been a very in-depth thread.

Thank you for your patience.

I think it is just in certain nuances that we disagree. I don't like disagreeing with you, I have such respect for your views on these things that it feels like cognitive dissonance.

Incidentally - the video of Callum isn't showing here. Could you check the link?

Edit: It's showing now.

Pages